matheraum.de
Raum für Mathematik
Offene Informations- und Nachhilfegemeinschaft

Für Schüler, Studenten, Lehrer, Mathematik-Interessierte.
Hallo Gast!einloggen | registrieren ]
Startseite · Forum · Wissen · Kurse · Mitglieder · Team · Impressum
Forenbaum
^ Forenbaum
Status Mathe
  Status Schulmathe
    Status Primarstufe
    Status Mathe Klassen 5-7
    Status Mathe Klassen 8-10
    Status Oberstufenmathe
    Status Mathe-Wettbewerbe
    Status Sonstiges
  Status Hochschulmathe
    Status Uni-Analysis
    Status Uni-Lin. Algebra
    Status Algebra+Zahlentheo.
    Status Diskrete Mathematik
    Status Fachdidaktik
    Status Finanz+Versicherung
    Status Logik+Mengenlehre
    Status Numerik
    Status Uni-Stochastik
    Status Topologie+Geometrie
    Status Uni-Sonstiges
  Status Mathe-Vorkurse
    Status Organisatorisches
    Status Schule
    Status Universität
  Status Mathe-Software
    Status Derive
    Status DynaGeo
    Status FunkyPlot
    Status GeoGebra
    Status LaTeX
    Status Maple
    Status MathCad
    Status Mathematica
    Status Matlab
    Status Maxima
    Status MuPad
    Status Taschenrechner

Gezeigt werden alle Foren bis zur Tiefe 2

Navigation
 Startseite...
 Neuerdings beta neu
 Forum...
 vorwissen...
 vorkurse...
 Werkzeuge...
 Nachhilfevermittlung beta...
 Online-Spiele beta
 Suchen
 Verein...
 Impressum
Das Projekt
Server und Internetanbindung werden durch Spenden finanziert.
Organisiert wird das Projekt von unserem Koordinatorenteam.
Hunderte Mitglieder helfen ehrenamtlich in unseren moderierten Foren.
Anbieter der Seite ist der gemeinnützige Verein "Vorhilfe.de e.V.".
Partnerseiten
Dt. Schulen im Ausland: Mathe-Seiten:Weitere Fächer:

Open Source FunktionenplotterFunkyPlot: Kostenloser und quelloffener Funktionenplotter für Linux und andere Betriebssysteme
StartseiteMatheForenUni-NumerikDiskrete Fouriertransformation
Foren für weitere Schulfächer findest Du auf www.vorhilfe.de z.B. Informatik • Physik • Technik • Biologie • Chemie
Forum "Uni-Numerik" - Diskrete Fouriertransformation
Diskrete Fouriertransformation < Numerik < Hochschule < Mathe < Vorhilfe
Ansicht: [ geschachtelt ] | ^ Forum "Uni-Numerik"  | ^^ Alle Foren  | ^ Forenbaum  | Materialien

Diskrete Fouriertransformation: Frage (beantwortet)
Status: (Frage) beantwortet Status 
Datum: 16:48 Mo 02.12.2019
Autor: inkeddude

Eingabefehler: "{" und "}" müssen immer paarweise auftreten, es wurde aber ein Teil ohne Entsprechung gefunden (siehe rote Markierung)

Aufgabe
Sei $n \in \mathbb{N}_{0}$ und $ k \in \{ 0, 1, \ldots, n \}$. Gegeben seien Punktepaare $\{(x_{j}, y_{j})_{j = 0}^{n}$ mit äquidistanten Knoten $x_{j} = \frac{2 \pi j}{n + 1}$



a) Zeigen Sie, dass die diskrete Fouriertransformation $F_{n} : \mathbb{C}^{ n + 1} \rightarrow \mathbb{C}^{ n + 1}, die durch $(i = \sqrt{ - 1})$

$(F_{n} (y))_{k} := \frac{1}{n + 1} \sum\limits_{j = 0}^{n} y_{j} e^{- i j x_{k}}$ definiert ist, folgende Eigenschaft erfüllt:


$\vert \vert F_{n}(y) \vert\vert_{2} = \vert \vert \frac{1}{\sqrt{n + 1}} y \vert\vert_{2}\quad \forall y \in \mathbb{C}^{ n+ 1}$



b) Die Faltung $*:  \mathbb{C}^{ n + 1} \times \mathbb{C}^{ n + 1} \rightarrow \mathbb{C}^{ n + 1}$ von zwei Vektoren $y, z \in \mathbb{C}^{ n + 1}$ (die in beiden Indexrichtungen periodisch fortgesetzt sind) sind definiert durch


$(y * z)_{k} := \sum\limits_{ j = 0}^{n} y_{k - j} z_{j}$   $(k \in \{0, 1, \ldots, n \})$.




Zeigen Sie: Für alle $y, z \in \mathbb{C}^{ n + 1}$ gilt



$\frac{1}{n + 1} F_{n}(y * z) = (F_{n}(y)) \cdot ' (F_{n}(z))$,


wobei $\cdot '$ die punktweise Multiplikation ist, die durch $(y \cdot' z)_{k} :=  y_{k } z_{k}$ definiert ist.

Ich bin ein wenig verzweifelt, da morgen  früh schon die Abgabe ist und ich bis jetzt immer noch nicht weiß, wo der Fehler in meiner Rechnung ist, um die a) zu lösen.

Mein Ansatz zu $a)$ ist:

$F_{n}(y)$ ist ja ein $n +1$ - dimensionaler Vektor, also $F_{n}(y) =  \left( \begin{array}{c} (F_{n}(y))_{0} \\\ \vdots \\\ (F_{n}(y))_{n}  \end{array}\right) $


Betrachten wir die $k$ -te Komponente. Es ist


$ (F_{n}(y))_{k} = \frac{1}{n +1} \sum\limits_{j = 0}^{n} y_{j} e^{- i j x_{k}} = \frac{1}{n +1} \sum\limits_{j = 0}^{n} y_{j} e^{- i j \frac{2 \pi k}{n + 1}} =  \frac{1}{n +1} \sum\limits_{j = 0}^{n} y_{j} \frac{1}{\sqrt[n + 1]{e^{i j 2 \pi k}}} = \frac{1}{n +1} \sum\limits_{j = 0}^{n} y_{j} \frac{1}{\sqrt[n + 1]{cos(j 2 \pi k) + i sin(j 2 \pi k)}} = \frac{1}{n +1} \sum\limits_{j = 0}^{n} y_{j}$



Also ist $(F_{n}(y))_{k} = \frac{1}{n +1} \sum\limits_{j = 0}^{n} y_{j}\quad \forall k \in \{0, 1, \ldots, n\}$


Damit haben wir  $F_{n}(y) =  \left( \begin{array}{c} \frac{1}{n +1} \sum\limits_{j = 0}^{n} y_{j} \\\ \vdots \\\ \frac{1}{n +1} \sum\limits_{j = 0}^{n} y_{j}\end{array}\right) $



Wenn wir die euklidische Norm bilden, erhalten wir:


$\vert \vert F_{n}(y) \vert \vert_{2} =  \left ( \left ( \frac{1}{n + 1} \sum\limits_{j = 0}^{n} y_{j} \right )^{2} + \ldots + \left ( \frac{1}{n + 1} \sum\limits_{j = 0}^{n} y_{j} \right )^{2}  \right )^{\frac{1}{2}} = \left ( (n + 1) \cdot \left ( \frac{1}{n + 1} \sum\limits_{j = 0}^{n} y_{j} \right )^{2} \right )^{\frac{1}{2}}$

$ = \sqrt{n + 1} \cdot \frac{1}{n + 1} \sum\limits_{j = 0}^{n} y_{j} =
\frac{1}{\sqrt{n + 1}} \cdot  \sum\limits_{j = 0}^{n} y_{j}$


Aber das ist ja nicht das selbe wie $\vert \vert \frac{1}{\sqrt{n + 1}} y \vert \vert_{2}$


Kann mir da jemand helfen und mir evtl. eine  Tipp bei der b) geben ? Das wäre echt super!



lg, inkeddude



Ich habe diese Frage in keinem Forum auf anderen Internetseiten gestellt.


        
Bezug
Diskrete Fouriertransformation: Antwort
Status: (Antwort) fertig Status 
Datum: 18:04 Mo 02.12.2019
Autor: Gonozal_IX

Hiho,
  

> Mein Ansatz zu [mm]a)[/mm] ist:

> Betrachten wir die [mm]k[/mm] -te Komponente. Es ist
> [mm](F_{n}(y))_{k} = \frac{1}{n +1} \sum\limits_{j = 0}^{n} y_{j} e^{- i j x_{k}} = \frac{1}{n +1} \sum\limits_{j = 0}^{n} y_{j} e^{- i j \frac{2 \pi k}{n + 1}} = \frac{1}{n +1} \sum\limits_{j = 0}^{n} y_{j} \frac{1}{\sqrt[n + 1]{e^{i j 2 \pi k}}} = \frac{1}{n +1} \sum\limits_{j = 0}^{n} y_{j} \frac{1}{\sqrt[n + 1]{cos(j 2 \pi k) + i sin(j 2 \pi k)}} = \frac{1}{n +1} \sum\limits_{j = 0}^{n} y_{j}[/mm]Eingabefehler: "\left" und "\right" müssen immer paarweise auftreten, es wurde aber ein Teil ohne Entsprechung gefunden (siehe rote Markierung)

Eingabefehler: "\left" und "\right" müssen immer paarweise auftreten, es wurde aber ein Teil ohne Entsprechung gefunden (siehe rote Markierung)



Dein letztes Gleichheitszeichen ist falsch.
Letztendlich behauptest du ja durch deine Umformungsschritte: $e^{- i j \frac{2 \pi k}{n + 1}}  = 1$  für beliebige $j,k,n$

Das stimmt natürlich nicht. Du befindest dich in $\IC$, da ist Wurzelziehen keine eindeutige Operation…

Der Beweis zur a) geht eigentlich viel schneller, wenn du die Norm-Definition mal sauber hinschreibst, wir haben nämlich (mal mit Quadraten um uns die Wurzeln zu sparen)

$ \vert \vert F_{n}(y) \vert\vert_{2}^2 = \sum_{k=0}^n \left| (F_{n} (y))_{k} \right|^2$

Aber nun gilt:
$\left| (F_{n} (y))_{k} \right|^2 = (F_{n} (y))_{k} \cdot \overline{(F_{n} (y))_{k}} =  \left(\frac{1}{n + 1} \sum\limits_{j = 0}^{n} y_{j} e^{- i j x_{k}}\right) \cdot \left( \frac{1}{n + 1} \sum\limits_{j = 0}^{n} y_{j} e^{i j x_{k}}\right) = \frac{1}{(n + 1)^2} \sum\limits_{j = 0}^{n} \left(y_j^2 + \sum_{m \not= j} y_jy_m\left(e^{-i(j-m)x_k} + e^{i(j-m)x_k}\right)\right)$

Beim letzten Gleichheitszeichen musst du dir klar machen, dass beim Produkt der Summen nur bei den Elemente mit demselben Index das $e^{i\varphi}$ gerade kürzt. Bei den Elementen mit gemischten Indizes erhalten wir denselben Term jeweils mit unterschiedlichem Vorzeichen im Exponenten.

Summieren wir das nun über alle Komponenten, so erhalten wir:

$ \vert \vert F_{n}(y) \vert\vert_{2}^2 = \sum_{k=0}^n \left| (F_{n} (y))_{k} \right|^2 = \frac{1}{n+1}\sum_{j=0}^{n}y_j^2 +  \frac{1}{(n+1)^2} \sum_{k=0}^n \sum_{j=0}^{n} \sum_{m \not= j} y_jy_m\left(e^{-i(j-m)x_k} + e^{i(j-m)x_k}\right)\right)$

$= \left|\left|\frac{1}{\sqrt{n + 1}} y \right|\right|_2^2 + C$

mit $C =  \frac{1}{(n+1)^2} \sum_{k=0}^n \sum_{j=0}^{n} \sum_{m \not= j} y_jy_m\left(e^{-i(j-m)x_k} + e^{i(j-m)x_k}\right)\right)$

Jetzt überlege dir mal, wie du zeigen kannst, dass $C = 0$ gilt.

Wenn du das verstanden hast, widmen wir uns der b)

Gruß,
Gono

Bezug
Ansicht: [ geschachtelt ] | ^ Forum "Uni-Numerik"  | ^^ Alle Foren  | ^ Forenbaum  | Materialien


^ Seitenanfang ^
www.matheraum.de
[ Startseite | Forum | Wissen | Kurse | Mitglieder | Team | Impressum ]